a我考网

 找回密码
 立即注册

QQ登录

只需一步,快速开始

扫一扫,访问微社区

查看: 140|回复: 0

LSAT考试全真题二SECTION1(2)

[复制链接]
发表于 2012-8-15 13:24:23 | 显示全部楼层 |阅读模式
Question 7-12, L" m& ^* Q! k/ L8 k. Z7 w9 |

% E0 [8 F. p! D0 j& p3 qFour lions-F, G, H, J-and two tigers-K and M-will be assigned to exactly six stalls, one animal per stall. The stalls are arranged as follows:
) r7 M, k! [; i4 p# Y$ ]. d7 K- Z# ~
 First Row:  1  2  3) H, D' \1 O9 x, g. [
8 I% f4 T* g/ E- B3 U: m5 W
 Second Row:  4  5  6
9 k6 O& {) q" V- V
4 G) W0 ?' {) v7 S3 F2 XThe only stalls that face each other are stalls 1 and 4, stalls 2 and 5, and stalls 3 and 6. The following conditions apply:1 B3 P' L3 Q4 m- O

$ `. v. u8 b3 ?) @4 E The tigers' stalls cannot face each other.% |% F+ c6 a% \3 w& S

9 l$ N# y2 O( {. c A lion must be assigned to stall 1' G8 W: d: q9 T! `
 $ N# i/ l, S, c8 U# h9 [  E& ?) ^
 H must be assigned to stall 6.
- J# F! d: y* Q3 r! ~
5 T0 `2 C: H: I" f: _ J must be assigned to a stall numbered one higher than K's stall.. Z  f4 h; s* Q5 u2 o
9 v" s) _3 Q+ R  a- E% z/ N
 K cannot be assigned to the stall that faces H's stall.
7 H( Q- M/ k$ F* {1 \! N
& A+ {$ z. J/ q2 ?& o7. Which one of the following must be true?( @1 D' X' m1 y* j1 Z5 K

' C9 U( q: x" Y% J2 i: J3 N5 j (A) F is assigned to an even-numbered stall
3 g8 s1 X& C! z$ ^; U (B) F is assigned to stall 13 O, l- `; \9 i4 X8 c5 ?
 (C) J is assigned to stall 2 or else stall 3
" j, L$ P" `$ f( e (D) J is assigned to stall 3 or else stall 4
2 b6 q- a$ R- P (E) K is assigned to stall 2 or else stall 4% p6 m0 l6 A3 R

" L4 X# S' \& K! D" ]* ]8 l8. Which one of the following could be true?
# O( S% D. r8 {; i5 ]
' p' m0 l5 C% R, H (A) F's stall is numbered one higher than J's stall( v" Y% ]# [  d. f1 P/ k
 (B) H's stall faces M's stall/ C8 R, v# p/ H  E" t
 (C) J is assigned to stall 4# F3 R1 c, O  n% ^5 }! E& ?
 (D) K's stall faces J'S stall
- j8 J: n5 K0 v; C: X& B (E) K's stall is in a different row than J's stall5 J' `1 \" ]  T1 `
2 I: B4 V0 X/ R- i0 {+ e; e( t
9. Which one of the following must be true?
" n6 M4 i; K& z8 \( m4 Q1 {. M) `& R: I: N4 D
 (A) A tiger is assigned to stall 2$ U* A2 U7 S' b" {6 H, y. K& ]
 (B) A tiger is assigned to stall 58 C+ j' L0 L2 n' _; Z
 (C) K's stall is in a different row from M's stall
  j+ v% |& U* j/ U, U1 D (D) Each tiger is assigned to an even-numbered stall* R6 O' o4 G' e: w  z: Q
 (E) Each lion is assigned to a stall that faces a tiger is stall
% _) p/ R: P7 Q4 a
& j: U  j7 {' r( j* h10. If K's stall is in the same row as H's stall which one of the following must be true?) _/ B7 p4 E3 H0 v+ ]1 @( I! L
( Z; w$ k$ O9 ^1 ~* Y. r* c
  (A) F's stall is in the same row as J's stall4 c" g$ \* j& T; v  M4 }
  (B) F is assigned to a lower-numbered stall than G$ @1 D: r) b3 a. \. v
  (C) G is assigned to a lower-numbered stall than M
3 g% K( z" Q: v" }$ o  (D) G's stall faces H's stall
; [$ k  n5 X; S$ g# L2 z0 O, y  (E) M's stall is in the same row as G's stall
2 k4 ?' r+ ?: G# T: D/ H) @" V- V" h
11. If J is assigned to stall 3, which one of the following could be true?
$ Q6 E) m$ i" o8 W+ P; s$ \# D4 t
  (A) F is assigned to stall 22 |" r' J6 C; X& B2 m$ X3 L1 r
  (B) F is assigned to stall 41 [+ f- @: J# T, M
  (C) G is assigned to stall 1
) h' }3 Y* k% o! Y  (D) G is assigned to stall 46 \& W6 Z; x( o
  (E) M is assigned to stall 5
( ^9 O4 F% v% g$ K) b: C' K' E2 }5 P) u- V2 y! b
12. Which one of the following must be true5 s  ?4 z: o: ^( r# T1 H7 h! g( @
# A* s2 x0 l% K- i# \4 q
  (A) A tiger is assigned to stall 2' Z% i  B7 M# K& j2 q
  (B) A tiger is assigned to stall 4
3 e! p8 M% h: i. A+ x  (C) A tiger is assigned to stall 5
' v7 a$ Y2 I1 h0 J3 Y+ s9 k  (D) A lion is assigned to stall 3
& Z0 b2 e2 B, V  (E) A lion is assigned to stall 4
回复

使用道具 举报

您需要登录后才可以回帖 登录 | 立即注册

本版积分规则

Archiver|手机版|小黑屋|Woexam.Com ( 湘ICP备18023104号 )

GMT+8, 2024-5-6 20:09 , Processed in 0.187823 second(s), 21 queries .

Powered by Discuz! X3.4 Licensed

© 2001-2017 Comsenz Inc.

快速回复 返回顶部 返回列表